LSAT and Law School Admissions Forum

Get expert LSAT preparation and law school admissions advice from PowerScore Test Preparation.

 Administrator
PowerScore Staff
  • PowerScore Staff
  • Posts: 8917
  • Joined: Feb 02, 2011
|
#36418
Complete Question Explanation

Resolve the Paradox Except. The correct answer choice is (A)

The stimulus presents an apparent paradox, so you must be aware of the contrasting elements:

..... 1. Tuition per class has not risen over a four-year period.
..... 2. Within that period, many students who once could afford tuition cannot now.

There are many ways to resolve this paradox. First of all, the stimulus leaves you a clue when it
refers to tuition per class. Perhaps the students are taking more classes now, which would make their
tuition rise even though tuition per class is constant. Second, there are many other factors—including
income, scholarships, and living costs—that play into whether a student can afford even a constant
tuition.

Since the question is a Resolve EXCEPT question, you should eliminate the four choices that resolve
the paradox, and select the choice that does not resolve the paradox.

Answer choice (A): This is the correct answer choice. This answer addresses teacher salaries,
and teacher salaries do not affect the ability of students to afford tuition when the tuition per class is
constant.

Answer choice (B): This choice resolves the paradox by pointing out that full-time students are
required to enroll in more classes, and thus they pay more money this year (remember, the stimulus
referenced per-class tuition as being constant).

Answer choice (C): This choice resolves the paradox by establishing that living costs are rising,
which would mean that the students have less money to spend on tuition.

Answer choice (D): This response resolves the paradox in very sly fashion. Even though it defi nitely
does not establish that a signifi cant number of students fail to keep their scholarships, it does
suggest that a great number of the students would initially be able to afford tuition because of the
scholarships, and establish that it is possible to lose that scholarship, somewhat reducing one’s
ability to afford tuition.

Answer choice (E): This choice resolves the paradox by suggesting that many students no longer
have work-study jobs available. Those students who needed the part-time offi ce jobs might fi nd
employment elsewhere, but cutting the students out of the university jobs would have some effect on
student income, and therefore their ability to afford tuition. For instance, the university town could
be small and unable to support the students as part-time employees, working in-town could present
confl icts with classes, or non-university jobs might tend to pay much less.
 8scn
  • Posts: 18
  • Joined: Nov 21, 2011
|
#3139
Why is A correct and not D? I thought A does resolve the discrepancy because if faculty salaries have risen, it could be that tuition has to increase to fund the faculty’s increased salaries.

For D, I thought it was reasonable to assume that the university didn’t just start renewing scholarships when the students in the stimulus first enrolled (ie. the university has been renewing scholarships for a long time before the students enrolled). So, when the students first enrolled, scholarships would still have been renewed for students of previous years (who were considered new to the university in previous years) who maintained high grade averages and qualified for the renewals. This doesn’t really explain the increase in tuition fees, since the costs of scholarship renewals have always occurred.
 Steve Stein
PowerScore Staff
  • PowerScore Staff
  • Posts: 1153
  • Joined: Apr 11, 2011
|
#3142
Thanks for your question. In this one the stimulus presents an apparent paradox, so you must be aware of the contrasting elements:

1. Tuition per class has not risen over a four-year period.
2. Within that period, many students who once could afford tuition cannot now.

There are many ways to resolve this paradox. First of all, the stimulus leaves you a clue when it refers to tuition per class. Perhaps the students are taking more classes now, which would make their tuition rise even though tuition per class is constant. Second, there are many other factors—including income, scholarships, and living costs—that play into whether a student can afford even a constant tuition.

Since the question is a Resolve EXCEPT question, you should eliminate the four choices that resolve the paradox, and select the choice that does not resolve the paradox.

Correct answer choice A addresses teacher salaries, and teacher salaries do not affect the ability of students to afford tuition when the tuition per class is constant.

As for answer choice D, that one resolves the paradox in very sly fashion. Even though it definitely does not establish that a significant number of students fail to keep their scholarships, it does suggest that a great number of the students would initially be able to afford tuition because of the scholarships, and establish that it is possible to lose that scholarship, somewhat reducing one’s ability to afford tuition.


Let me know whether this is clear--thanks!
 suburbsinmymindseye
  • Posts: 8
  • Joined: Jan 03, 2020
|
#76917
The LSAT's insistence on using precise wording to describe answers kind of bothers me when I feel like they don't adhere to their own principles.

Answer choice D simply states that these scholarships are renewed for students with high GPAs. It doesn't state that the scholarship is ONLY renewed for students with high GPAs. It's logically inaccurate to say that we can draw the conclusion that students lose their scholarship every year. I feel like this answer is requiring us to draw an assumption that we shouldn't have to make -- just like answer choice A.
 Frank Peter
PowerScore Staff
  • PowerScore Staff
  • Posts: 99
  • Joined: May 14, 2020
|
#77137
Hi Suburbs,

The level of precision in word choice often depends on the type of question (for example, a stimulus relying on conditional reasoning requires us to pay very close attention to each word), and the general scope of the stimulus (a stimulus worded very broadly, versus one that is worded more narrowly).

With regard to (D), I think your reading of it may be a little too rigid given the type of question, and the general directive in the section instructions to not make assumptions that "are by commonsense standards implausible." If we're told "a large number of scholarships that are renewed each year for the students who maintain high grade averages", the commonsense assumption would be that students who do not maintain a high GPA will not get to keep their scholarship.

(A) is the correct answer because it doesn't help resolve the paradox, since it doesn't explain why many students would suddenly not be able to afford tuition, even though the cost per course has not risen.

Get the most out of your LSAT Prep Plus subscription.

Analyze and track your performance with our Testing and Analytics Package.